Measuring the polariztion of a pair of entangled photons

In summary, the conversation discusses the experiment of two entangled photons passing through linear polarizers and detectors. The apparatus connected to the detectors should be in a superposition of detecting one or both photons. The method used to find the coefficients is described, using the eigenvectors and observables associated with the polarizers. The resulting amplitudes are then used to explain the correlation between the two detectors, which is found to be sinusoidal. The question is raised about the validity of the argument, as the eigenvectors may not accurately represent the state of the system.
  • #1
Cylten
1
0
Hi,

I'm quite new to quantum mechanics, learning about it in my free time in a life-long learning fashion :) I've been trying to find a solution to a problem for some time, and the results I included below appear to be OK, but I have doubts about the method I used, so any help or guidance would be much appreciated.

I imagine the following experiment: suppose there is a source that emits two entangled photons along the z axis, A and B, both linearly polarized, but in an orthogonal direction. Photon A encounters a linear polarizer aligned with the x and y axes and a detector behind it. Photon B encounters a linear polarizer rotated by ##\phi## related to the first one and another detector behind it. There is some apparatus, C, connected to both detectors. I think after the experiment, the apparatus should be in a superposition of detecting one, the other, both, or neither photons:
$$ |C\rangle = x\left|C_{A\&B}\right> + y\left|C_A\right>+ z\left|C_B\right>+ w\left|C_0\right>. $$
I'm trying to find the coefficients (amplitudes) x, y, z and w, and I know the order in which the photons interact with the polarizers and detectors should not matter.

I've tried various ways to do this, and the most promising was the following:

In isolation, the polarization of photon A can be described in the 2-dimensional Hilbert space ##H_A##. Let the observable associated with the x,y-aligned polarizer be P with the following eigenvectors:
$$ P = \begin{pmatrix}1&0\\ 0&-1\end{pmatrix}
~~~~~|+p\rangle = \begin{pmatrix}1\\0\end{pmatrix}
~~~~~|-p\rangle = \begin{pmatrix}0\\1\end{pmatrix}. $$
The polarization of photon B can be described in a similar space ##H_B##. Let the observable associated with the ##\phi##-rotated polarizer be ##P_\phi## with the following eigenvectors (using the equivalents of ##|+p\rangle## and ##|-p\rangle## as the basis):
$$ P_\phi = \begin{pmatrix}\cos2\phi& \sin2\phi\\ \sin2\phi& -\cos2\phi\end{pmatrix}
~~~~~|+\phi\rangle = \begin{pmatrix}\cos\phi\\ \sin\phi\end{pmatrix}
~~~~~|-\phi\rangle = \begin{pmatrix}-\sin\phi\\ \cos\phi\end{pmatrix}. $$

The (linear) polarization of the entangled photons, if A is polarized in the ##a## direction, can be described in ##H_A\otimes H_B## as
$$ |\Psi\rangle = \cos a |+p\rangle\otimes|-p\rangle + \sin a |-p\rangle\otimes|+p\rangle, $$
and, using the ##|\pm p\rangle \otimes|\pm p\rangle## basis, this can be expressed as
$$ |\Psi\rangle = \begin{pmatrix}0\\ \cos a\\ \sin a\\ 0\end{pmatrix}. $$
Now in order to find the outcome of the experiment, I considered ##P\otimes P_\phi## as an observable in ##H_A\otimes H_B## (tensor product of Hermitians is Hermitian). Its eigenvectors are the tensor products of the eigenvectors of ##P## and ##P_\phi##, which are:
$$ \begin{aligned}
|+p\rangle\otimes|+\phi\rangle =& \begin{pmatrix}\cos\phi\\ \sin\phi\\ 0\\ 0\end{pmatrix} \\
|+p\rangle\otimes|-\phi\rangle =& \begin{pmatrix}-\sin\phi\\ \cos\phi\\ 0\\ 0\end{pmatrix} \\
|-p\rangle\otimes|+\phi\rangle =& \begin{pmatrix}0\\ 0\\ \cos\phi\\ \sin\phi\end{pmatrix} \\
|-p\rangle\otimes|-\phi\rangle =& \begin{pmatrix}0\\ 0\\ -\sin\phi\\ \cos\phi\end{pmatrix}
\end{aligned} $$

And from these, the amplitudes appear the be:
$$ \begin{aligned}
\langle+p+\phi|\Psi\rangle = \sin\phi\cos a &= x \\
\langle+p-\phi|\Psi\rangle = \cos\phi\cos a &= y \\
\langle-p+\phi|\Psi\rangle = \cos\phi\sin a &= z \\
\langle-p-\phi|\Psi\rangle = -\sin\phi\sin a &= w,
\end{aligned} $$
as we expect both photons to go through the polarizers at ##\phi=\pi/2## and ##a=0##.

This result also seems to be in accordance with the reasoning around Bell's theorem,
which, as I understand it, suggests that the average correlation between the two detectors, defined as
$$ \text{Cor} = \frac{
\left(\begin{matrix}\text{number of experiments}\\ \text{showing correlation}\end{matrix}\right)
- \left(\begin{matrix}\text{number of experiments}\\ \text{with no correlation}\end{matrix}\right) }
{\text{number of experiments}}. $$
as a function of ##\phi## is not linear, but sinusoid. Based on the above amplitudes, this is
$$ \begin{aligned} \text{Cor} = \sin^2\phi\cos^2a + \sin^2\phi\sin^2a - \cos^2\phi\cos^2a - \cos^2\phi\sin^2a &\\
= \sin^2\phi - \cos^2\phi = -\cos(2\phi)&, \end{aligned}$$
which seems to be OK as we expect complete correlation at ##\phi=\pi/2## (when the polarizers are aligned orthogonally) and again at ##\phi=3\pi/2##.

Does this mean that the above values of x,y,z and w are correct? What I find most confusing is that the eigenvectors of ##P\otimes P_\phi## are (I think) not states the system can actually be in, as, for example, ##|\Psi\rangle = |+p\rangle\otimes |+\phi\rangle## would mean that A is polarized in the ##|+p\rangle## direction and B is polarized in the ##|+\phi\rangle## direction, which seems to contradict the entanglement. Can this be fixed or does this mean that the whole argument is invalid?

Many thanks in advance!
 
Physics news on Phys.org
  • #2
once you measured the system it is no more entangled and become a separable state namely one of the eigenstates with the corresponding probabilities
 

Related to Measuring the polariztion of a pair of entangled photons

1. What is the significance of measuring the polarization of entangled photons?

The polarization of entangled photons is significant because it allows us to observe the phenomenon of quantum entanglement. This is a phenomenon where two particles are connected in such a way that, even when separated by large distances, the state of one particle can instantly affect the state of the other particle. Measuring the polarization of entangled photons can provide valuable insights into the nature of quantum mechanics and potentially lead to advancements in technologies such as quantum computing.

2. How is the polarization of entangled photons measured?

The polarization of entangled photons is typically measured using a polarizer, which is a device that can filter light based on its polarization. The photons are passed through the polarizer and the resulting output is measured using a detector. This allows us to determine the polarization state of the photons and observe any correlations between the entangled particles.

3. Can the polarization of entangled photons be changed?

Yes, the polarization of entangled photons can be changed. This is because the polarization state of a photon is not a fixed property, but rather can be influenced by external factors such as a polarizer. However, changing the polarization of one entangled photon will also affect the polarization of its entangled partner, as they are connected through quantum entanglement.

4. What are some practical applications of measuring the polarization of entangled photons?

Measuring the polarization of entangled photons has potential applications in fields such as quantum cryptography, quantum teleportation, and quantum communication. These technologies rely on the unique properties of quantum entanglement and are currently being researched and developed for various practical purposes.

5. Is measuring the polarization of entangled photons a difficult process?

Measuring the polarization of entangled photons can be challenging due to the delicate and complex nature of quantum systems. It requires specialized equipment and techniques, and the results can be affected by external factors such as noise and interference. However, advancements in technology and techniques have made it possible to accurately measure the polarization of entangled photons, leading to further understanding and potential breakthroughs in the field of quantum mechanics.

Similar threads

Replies
21
Views
2K
Replies
2
Views
830
  • Quantum Physics
Replies
26
Views
1K
  • Quantum Physics
Replies
1
Views
771
  • Quantum Physics
Replies
2
Views
833
  • Quantum Physics
Replies
1
Views
981
Replies
3
Views
481
Replies
5
Views
1K
Replies
2
Views
726
  • Quantum Physics
Replies
1
Views
1K
Back
Top